Not able to solve a matrix problem












1












$begingroup$


This is my first question of SO so sorry if I made a mistake somewhere.



I have this matrices problem which I'm not able to solve. I asked many of my friends but they just don't get the question.
This is the question:



$text{Let n} geq 3. text{Let A} = ((a_{ij}))_{1 leq i, j leq n} text{be an n} times text{n matrix such that a}_{ij} in {1, -1} text{for all 0} leq i, j leq n.
text{Suppose a}_{k1} = text{1 for all 1} leq k leq text{n. } \ text{And} sum_{k = 1}^{n} a_{ki}cdot a_{kj}text{= 0. For all i} neq j.\ text{Show that n is a multiple of 4.}$



(sorry I can't add an image and I'm not so good at latex too)



I think I've understood the summation part: when k = 1, the argument of summation becomes $a_{1i} cdot a_{1j}$. Which is the sum of product of elements of 1st row taken 2 at a time. (that's what I interpreted $a_{1i} cdot a_{1j}$ as. Is this right?)
I'm not able to utilize the summation to get an equation in n.



Thanx in advance!










share|cite|improve this question









$endgroup$












  • $begingroup$
    You are looking at Hadamard matrices, so now you know what to look for.
    $endgroup$
    – Gerry Myerson
    Nov 30 '18 at 11:54










  • $begingroup$
    See, e.g., math.stackexchange.com/questions/753867/…
    $endgroup$
    – Gerry Myerson
    Nov 30 '18 at 11:59
















1












$begingroup$


This is my first question of SO so sorry if I made a mistake somewhere.



I have this matrices problem which I'm not able to solve. I asked many of my friends but they just don't get the question.
This is the question:



$text{Let n} geq 3. text{Let A} = ((a_{ij}))_{1 leq i, j leq n} text{be an n} times text{n matrix such that a}_{ij} in {1, -1} text{for all 0} leq i, j leq n.
text{Suppose a}_{k1} = text{1 for all 1} leq k leq text{n. } \ text{And} sum_{k = 1}^{n} a_{ki}cdot a_{kj}text{= 0. For all i} neq j.\ text{Show that n is a multiple of 4.}$



(sorry I can't add an image and I'm not so good at latex too)



I think I've understood the summation part: when k = 1, the argument of summation becomes $a_{1i} cdot a_{1j}$. Which is the sum of product of elements of 1st row taken 2 at a time. (that's what I interpreted $a_{1i} cdot a_{1j}$ as. Is this right?)
I'm not able to utilize the summation to get an equation in n.



Thanx in advance!










share|cite|improve this question









$endgroup$












  • $begingroup$
    You are looking at Hadamard matrices, so now you know what to look for.
    $endgroup$
    – Gerry Myerson
    Nov 30 '18 at 11:54










  • $begingroup$
    See, e.g., math.stackexchange.com/questions/753867/…
    $endgroup$
    – Gerry Myerson
    Nov 30 '18 at 11:59














1












1








1


1



$begingroup$


This is my first question of SO so sorry if I made a mistake somewhere.



I have this matrices problem which I'm not able to solve. I asked many of my friends but they just don't get the question.
This is the question:



$text{Let n} geq 3. text{Let A} = ((a_{ij}))_{1 leq i, j leq n} text{be an n} times text{n matrix such that a}_{ij} in {1, -1} text{for all 0} leq i, j leq n.
text{Suppose a}_{k1} = text{1 for all 1} leq k leq text{n. } \ text{And} sum_{k = 1}^{n} a_{ki}cdot a_{kj}text{= 0. For all i} neq j.\ text{Show that n is a multiple of 4.}$



(sorry I can't add an image and I'm not so good at latex too)



I think I've understood the summation part: when k = 1, the argument of summation becomes $a_{1i} cdot a_{1j}$. Which is the sum of product of elements of 1st row taken 2 at a time. (that's what I interpreted $a_{1i} cdot a_{1j}$ as. Is this right?)
I'm not able to utilize the summation to get an equation in n.



Thanx in advance!










share|cite|improve this question









$endgroup$




This is my first question of SO so sorry if I made a mistake somewhere.



I have this matrices problem which I'm not able to solve. I asked many of my friends but they just don't get the question.
This is the question:



$text{Let n} geq 3. text{Let A} = ((a_{ij}))_{1 leq i, j leq n} text{be an n} times text{n matrix such that a}_{ij} in {1, -1} text{for all 0} leq i, j leq n.
text{Suppose a}_{k1} = text{1 for all 1} leq k leq text{n. } \ text{And} sum_{k = 1}^{n} a_{ki}cdot a_{kj}text{= 0. For all i} neq j.\ text{Show that n is a multiple of 4.}$



(sorry I can't add an image and I'm not so good at latex too)



I think I've understood the summation part: when k = 1, the argument of summation becomes $a_{1i} cdot a_{1j}$. Which is the sum of product of elements of 1st row taken 2 at a time. (that's what I interpreted $a_{1i} cdot a_{1j}$ as. Is this right?)
I'm not able to utilize the summation to get an equation in n.



Thanx in advance!







matrices summation






share|cite|improve this question













share|cite|improve this question











share|cite|improve this question




share|cite|improve this question










asked Nov 30 '18 at 10:58









YashKandalkarYashKandalkar

82




82












  • $begingroup$
    You are looking at Hadamard matrices, so now you know what to look for.
    $endgroup$
    – Gerry Myerson
    Nov 30 '18 at 11:54










  • $begingroup$
    See, e.g., math.stackexchange.com/questions/753867/…
    $endgroup$
    – Gerry Myerson
    Nov 30 '18 at 11:59


















  • $begingroup$
    You are looking at Hadamard matrices, so now you know what to look for.
    $endgroup$
    – Gerry Myerson
    Nov 30 '18 at 11:54










  • $begingroup$
    See, e.g., math.stackexchange.com/questions/753867/…
    $endgroup$
    – Gerry Myerson
    Nov 30 '18 at 11:59
















$begingroup$
You are looking at Hadamard matrices, so now you know what to look for.
$endgroup$
– Gerry Myerson
Nov 30 '18 at 11:54




$begingroup$
You are looking at Hadamard matrices, so now you know what to look for.
$endgroup$
– Gerry Myerson
Nov 30 '18 at 11:54












$begingroup$
See, e.g., math.stackexchange.com/questions/753867/…
$endgroup$
– Gerry Myerson
Nov 30 '18 at 11:59




$begingroup$
See, e.g., math.stackexchange.com/questions/753867/…
$endgroup$
– Gerry Myerson
Nov 30 '18 at 11:59










1 Answer
1






active

oldest

votes


















1












$begingroup$

Clearly $n$ can't be odd as the inner product of the first column, which solely consists of ones, must be zero. Hence in every other column we must have an equal number of ones and minus ones, let's call that number $m$. (That is $n=2m$.)



Now consider two column except the first one. Have a look at the rows in the first column with element one (there are $m$ such rows). Let the number of ones in the corresponding rows in the second column be $k$, clearly $0leq kleq m$. Their product equals obviously $k-(m-k)=2k-m$. Similarly, the sum of the product of the other rows is $2k-m$ as well.



So in total the sum of all products is $4k-2m=2(2k-m)$. As that sum is zero, $m$ must be even, hence $n$ is divisible by $4$.






share|cite|improve this answer









$endgroup$













    Your Answer





    StackExchange.ifUsing("editor", function () {
    return StackExchange.using("mathjaxEditing", function () {
    StackExchange.MarkdownEditor.creationCallbacks.add(function (editor, postfix) {
    StackExchange.mathjaxEditing.prepareWmdForMathJax(editor, postfix, [["$", "$"], ["\\(","\\)"]]);
    });
    });
    }, "mathjax-editing");

    StackExchange.ready(function() {
    var channelOptions = {
    tags: "".split(" "),
    id: "69"
    };
    initTagRenderer("".split(" "), "".split(" "), channelOptions);

    StackExchange.using("externalEditor", function() {
    // Have to fire editor after snippets, if snippets enabled
    if (StackExchange.settings.snippets.snippetsEnabled) {
    StackExchange.using("snippets", function() {
    createEditor();
    });
    }
    else {
    createEditor();
    }
    });

    function createEditor() {
    StackExchange.prepareEditor({
    heartbeatType: 'answer',
    autoActivateHeartbeat: false,
    convertImagesToLinks: true,
    noModals: true,
    showLowRepImageUploadWarning: true,
    reputationToPostImages: 10,
    bindNavPrevention: true,
    postfix: "",
    imageUploader: {
    brandingHtml: "Powered by u003ca class="icon-imgur-white" href="https://imgur.com/"u003eu003c/au003e",
    contentPolicyHtml: "User contributions licensed under u003ca href="https://creativecommons.org/licenses/by-sa/3.0/"u003ecc by-sa 3.0 with attribution requiredu003c/au003e u003ca href="https://stackoverflow.com/legal/content-policy"u003e(content policy)u003c/au003e",
    allowUrls: true
    },
    noCode: true, onDemand: true,
    discardSelector: ".discard-answer"
    ,immediatelyShowMarkdownHelp:true
    });


    }
    });














    draft saved

    draft discarded


















    StackExchange.ready(
    function () {
    StackExchange.openid.initPostLogin('.new-post-login', 'https%3a%2f%2fmath.stackexchange.com%2fquestions%2f3019953%2fnot-able-to-solve-a-matrix-problem%23new-answer', 'question_page');
    }
    );

    Post as a guest















    Required, but never shown

























    1 Answer
    1






    active

    oldest

    votes








    1 Answer
    1






    active

    oldest

    votes









    active

    oldest

    votes






    active

    oldest

    votes









    1












    $begingroup$

    Clearly $n$ can't be odd as the inner product of the first column, which solely consists of ones, must be zero. Hence in every other column we must have an equal number of ones and minus ones, let's call that number $m$. (That is $n=2m$.)



    Now consider two column except the first one. Have a look at the rows in the first column with element one (there are $m$ such rows). Let the number of ones in the corresponding rows in the second column be $k$, clearly $0leq kleq m$. Their product equals obviously $k-(m-k)=2k-m$. Similarly, the sum of the product of the other rows is $2k-m$ as well.



    So in total the sum of all products is $4k-2m=2(2k-m)$. As that sum is zero, $m$ must be even, hence $n$ is divisible by $4$.






    share|cite|improve this answer









    $endgroup$


















      1












      $begingroup$

      Clearly $n$ can't be odd as the inner product of the first column, which solely consists of ones, must be zero. Hence in every other column we must have an equal number of ones and minus ones, let's call that number $m$. (That is $n=2m$.)



      Now consider two column except the first one. Have a look at the rows in the first column with element one (there are $m$ such rows). Let the number of ones in the corresponding rows in the second column be $k$, clearly $0leq kleq m$. Their product equals obviously $k-(m-k)=2k-m$. Similarly, the sum of the product of the other rows is $2k-m$ as well.



      So in total the sum of all products is $4k-2m=2(2k-m)$. As that sum is zero, $m$ must be even, hence $n$ is divisible by $4$.






      share|cite|improve this answer









      $endgroup$
















        1












        1








        1





        $begingroup$

        Clearly $n$ can't be odd as the inner product of the first column, which solely consists of ones, must be zero. Hence in every other column we must have an equal number of ones and minus ones, let's call that number $m$. (That is $n=2m$.)



        Now consider two column except the first one. Have a look at the rows in the first column with element one (there are $m$ such rows). Let the number of ones in the corresponding rows in the second column be $k$, clearly $0leq kleq m$. Their product equals obviously $k-(m-k)=2k-m$. Similarly, the sum of the product of the other rows is $2k-m$ as well.



        So in total the sum of all products is $4k-2m=2(2k-m)$. As that sum is zero, $m$ must be even, hence $n$ is divisible by $4$.






        share|cite|improve this answer









        $endgroup$



        Clearly $n$ can't be odd as the inner product of the first column, which solely consists of ones, must be zero. Hence in every other column we must have an equal number of ones and minus ones, let's call that number $m$. (That is $n=2m$.)



        Now consider two column except the first one. Have a look at the rows in the first column with element one (there are $m$ such rows). Let the number of ones in the corresponding rows in the second column be $k$, clearly $0leq kleq m$. Their product equals obviously $k-(m-k)=2k-m$. Similarly, the sum of the product of the other rows is $2k-m$ as well.



        So in total the sum of all products is $4k-2m=2(2k-m)$. As that sum is zero, $m$ must be even, hence $n$ is divisible by $4$.







        share|cite|improve this answer












        share|cite|improve this answer



        share|cite|improve this answer










        answered Nov 30 '18 at 12:40









        Michael HoppeMichael Hoppe

        10.8k31834




        10.8k31834






























            draft saved

            draft discarded




















































            Thanks for contributing an answer to Mathematics Stack Exchange!


            • Please be sure to answer the question. Provide details and share your research!

            But avoid



            • Asking for help, clarification, or responding to other answers.

            • Making statements based on opinion; back them up with references or personal experience.


            Use MathJax to format equations. MathJax reference.


            To learn more, see our tips on writing great answers.




            draft saved


            draft discarded














            StackExchange.ready(
            function () {
            StackExchange.openid.initPostLogin('.new-post-login', 'https%3a%2f%2fmath.stackexchange.com%2fquestions%2f3019953%2fnot-able-to-solve-a-matrix-problem%23new-answer', 'question_page');
            }
            );

            Post as a guest















            Required, but never shown





















































            Required, but never shown














            Required, but never shown












            Required, but never shown







            Required, but never shown

































            Required, but never shown














            Required, but never shown












            Required, but never shown







            Required, but never shown







            Popular posts from this blog

            Le Mesnil-Réaume

            Ida-Boy-Ed-Garten

            web3.py web3.isConnected() returns false always